You are having a birthday party and are inviting 6 friends. You have 9 cupcakes, and you are going to share the cupcakes fairly among you and your 6 friends.
Which equation describes how many cupcakes each of you will receive?

Answers

Answer 1

Answer:

split the other three in half

Step-by-step explanation:


Related Questions

increase 6800 by 12%

Answers

Answer:

Step-by-step explanation:

6,800, percentage increased by 12% (percent) of its value = 7,616  

20, percentage increased by 1,000% (percent) of its value = 220  

1,000,000, percentage increased by 0.49% (percent) of its value = 1,004,900

Answer:

7616

hope it helps

mark brainliest please

Given right triangle ABC with altitude BD drawn to hypotenuse AC

Answers

Given:

In right triangle ABC, BD is the altitude drawn to hypotenuse AC.

[tex]AD=20, DC=45[/tex]

To find:

The length of BD.

Solution:

According to geometric mean theorem of triangles, if an altitude is drawn to the hypotenuse of a triangle, then the altitude is the geometric mean of the segments of the hypotenuse.

Using geometric mean theorem of triangles, we get

[tex]BD^2=AD\times DC[/tex]

[tex]x^2=20\times 45[/tex]

[tex]x^2=900[/tex]

Taking square root on both sides, we get

[tex]x=\dfrac{900}[/tex]

[tex]x=30[/tex]

Therefore, the length of BD is 30 units and the correct option is C.

3x + 1 over 4y2

What is the value of the expression above when x = 3 and y = 4? You must show all work and calculations to receive full credit.

Answers

Answer:

5/32

Step-by-step explanation:

(3x+1)/4y^2

(3(3)+1)/[4(4)^2]

(3*3+1)/[4(4)(4)]

(9+1)/[4*16]

10/64

simplify

5/32

The value of the expression at x = 3 and y = 4 is  [tex]\frac{5}{32}[/tex]   .

Concept:Firstly we will convert the expression into numerical form.Then, we will substitute the value of x & y respectively in the expression and simplify it.

How to solve the given question?Converting the expression into numerical form.
∴ Let [tex]F(x,y) = \frac{3x+1}{4y^2}[/tex]
Substituting x = 3 and y = 4 in the given expression.
∴[tex]F(x = 3,y =4) = \frac{3(3)+1}{4(4)^2} = \frac{10}{64}[/tex]
∴ [tex]F(3,4) = \frac{5}{32}[/tex]

Thus, the value of the expression at x = 3 and y = 4 is  [tex]\frac{5}{32}[/tex]   .

Learn more about expressions here:

https://brainly.com/question/723406

#SPJ2

Find the area of this regular polygon. Round to the nearest tenth. 8.8 ft

Answers

Answer:

I don't really know but I'm pretty sure u can fix it and answer this question

A bacteria culture is growing at a rate of
r(t) = 7e^0.6t
thousand bacteria per hour after t hours. How much did the bacteria population increase during the first two hours? (Round your answer to three decimal places.)

Answers

Answer:

[tex]{ \bf{r(t) = 7e {}^{0.6t} }} \\ { \tt{r(2) = 7 {e}^{0.6 \times 2} }} \\ = { \tt{7 {e}^{1.2} }} \\ = 23.241 \: thiusand bacteria \: per \: hour[/tex]

23.2 bactéria that is growing I think

PLEASE HELP ME ASAP!​

Answers

Answer:

A

Step-by-step explanation:

A


This is the correct answer

Question 4 of 5
How many solutions does 8 + 6
=
– 4 + have?
A. Two solutions
B. One solution
C. No solutions
D. Infinitely many solutions
SUBMIT

Answers

Your answer would be “One solution”
Explanation:
You can switch what you do with the - 4
You can do
8 + 6 - 4 which is 10
Or
8 - 4 + 6 which would still give us 10

Eighty members of a bike club were asked whether they like touring bikes and whether they like mountain bikes. A total of 70 like touring bikes, 47 like mountain bikes, and 5 do not like either.

A 4-column table with 3 rows. The first column has no label with entries likes mountain bikes, does not like mountain bikes, total. The second column is labeled liking touring bikes with entries a, c, 70. The third column is labeled does not like touring bikes with entries b, 5, e. The fourth column is labeled total with entries 47, d, 80.

What are the correct values of a, b, c, d, and e?

a = 42, b = 28, c = 33, d = 5, e = 10
a = 42, b = 5, c = 28, d = 33, e = 10
a = 5, b = 42, c = 28, d = 10, e = 33
a = 5, b = 10, c = 28, d = 33, e = 42

Answers

Answer: Choice B

a = 42, b = 5, c = 28, d = 33, e = 10

====================================================

Explanation:

Refer to the diagram below. We have a table of values in which some values (if not most) are unknown. We have variables as placeholders until we can figure out which numbers replace them.

Along the bottom row, we see that 70+e = 80, which solves to e = 10. I subtracted 70 from both sides.

That must mean the answer is between choice A or choice B.

--------------------------

Since e = 10, and b+5 = e (third column), we can replace the 'e' with 10 to get the equation b+5 = 10. That solves to b = 5. This rules out choice A.

The final answer is choice B

---------------------------

If you want to keep going to find a, c and d, then let's solve a+b = 47 which is the same as a+5 = 47 after replacing letter b with 5.

a+5 = 47 solves to a = 42 after subtracting both sides by 5.

Then along the first column, we see that a+c = 70 which is the same as 42+c = 70. Isolating c gets us c = 28

Lastly, c+5 = d is shown along the second row. Plug in c = 28 to find that d = c+5 = 28+5 = 33

So overall we have:

a = 42, b = 5, c = 28, d = 33, e = 10

Answer:

b is correct

Step-by-step explanation:

Encontrar X..................

Answers

X is 51
Because I did this a few days bakc

Butch will miss an important TV program while taking his statistics exam, so he sets
both his VCRs to record it. The first one records 70% of the time, and the second one
records 60% of the time. What is the probability that he gets home after the exam and
finds? (We assume that events A and B are independent, so with P(A)=0.7 and
P(B) = 0.6 respectively.)
(a) No copies of his program?
(b) One copy of his program?

Answers

Answer:

b

Step-by-step explanation:

this because both VCSs were recording on the same time hence Butch will have two copies of record and the A vcr having an Extra of what B vcr will

Match each inequality to the number line that represents its solution

Answers

Answer:

x - 51 [tex]\leq[/tex] 43

x - 99 [tex]\leq[/tex] -104

75 < 69 - x

150 + x ≤ 144

Step-by-step explanation:

Instructions: Find the area of the sector. Round your answer to the nearest tenth.
I’ll mark brainliest please help

Answers

Answer:

75.7m^2

Step-by-step explanation:

[tex] \frac{30}{360} \times \pi {r}^{2} \\ \frac{30}{360} \times \frac{22}{7} \times {17}^{2} \\ \frac{1}{12} \times \frac{22}{7} \times 289 \\ 75.6904761905 = 75.7[/tex]

Describe, in your own words, what "convenience sampling" is. 1 point

Answers

Answer:

It is a type of non-probability sampling that involves the sample being drawn from that part of the population that is close at hand

|x+2|+4=11
A) x=5,-9

Answers

Answer:

the mod always gives positive answers

so the answer will be

Step-by-step explanation:

x+2+4=11

x+6=11

x=11-6

x=5

answer is x is 5

Answer:

x=5,9

Step-by-step explanation:

|x+2|+4=11

|x+2| = 7

next you get two answers by adding two or subtracting two. SO the right answer should be 5 and 9. A is wrong . I DONT SEE THE OTHER ANSWER CHOICES BUT THIS IS WHAT I GOT.

What base makes the equation below true?

Note: A number should be entered, not a word.

Answers

Answer:

4

Step-by-step explanation:

in base 4

21 = (2 * 4) + (1 * 1) = 9

There are 13 apples in a basket. 4 of these apples are green. The rest are red. what is the ratio?​

Answers

Answer:

9:4 4:13

Step-by-step explanation:

Answer:

a) 9:4  b) 13 : 9

Step-by-step explanation:

13 = total

4 = green

13-4   ( which Is 9) = red

Ratio of red apples to green apples?

9:4

RATIO of all apples to red

13: 9

help please step by step

Answers

Answer:

1. 33km

2. 1.2ft

3. 16in

4. 20.1ft

5. 7.2yd

6. 38mi

Step-by-step explanation:

1. 10+11+12=33km

2. 1/2(1.8+0.6)1=1.2ft

3. 5+5+3+3=16in

4. 4.1+2.7+5.4+7.9=20.1ft

5. 1.8+1.8+1.8+1.8=7.2yd

6. 12+7+12+7=38mi

Prove formally. Step by step with statements and reasons.

Answers

Answer:

K

K -> H

H

B -> ~H

H not B (sorry I couldn't type out the symbol)

~B

Step-by-step explanation:

Verbally, what's happening is this:

We have an assertion of K. Since K -> H (K implicates H), and B -> ~H (B implicates a negation of H), a negation of B (~B) is proven. (In this entire process, nothing is given about the symbols except K is true, hence the assertion, which then implicates that H is true and that B is negated since B implicates that negated H is true when in fact it is the opposite.)

Formally, the answer would be the following:

K

K -> H

H

B -> ~H

H not B (sorry I couldn't type out the symbol)

~B

Hope this helped!

In isosceles △HAM, m∡A =32°, . What is m∠H?

Answers

Answer:

This all I can do

Step-by-step explanation:

m∠F. 20 In the diagram below, LATE is an isosceles ... 32 In the diagram of JEA below, m∠JEA = 90 and m∠EAJ = 48. ... cylinder that has a height of 15 cm and a diameter of 12 cm? ... If m∠HAM = 12, what is m∠AMT ? 1) 12. 2) 78. 3) 84.

HW HELP ASAP PLZZZZZ

Answers

Answer:

3

Step-by-step explanation:

I think its 3 because when we add 2 to x the graphic must be continue two step to right

Graph the set on a number line. {-1/2, -1 3/4}

Answers

Answer:

D

Step-by-step explanation:

Each dot is 1/4 and we are going from 0 to -2

A sales analyst listed the probabilities of profits and losses in dollars for a certain company. Find the mean, µ for the probability distribution.

x P(x)
-500 0.076
-250 0.191
0 0.265
250 0.316
500 0.152

Answers

Answer:

The mean is [tex]\mu = 69.25[/tex]

Step-by-step explanation:

We are given the following distribution:

P(x = -500) = 0.076

P(x = -250) = 0.191

P(x = 0) = 0.265

P(x = 250) = 0.316

P(x = 500) = 0.152

Find the mean, µ for the probability distribution.

To find the mean, we multiply each outcome by its probability. So

[tex]\mu = -500*0.076 - 250*0.191 + 0*0.265 + 250*0.316 + 500*0.152 = 69.25[/tex]

The mean is [tex]\mu = 69.25[/tex]

A delivery truck manager takes a sample of 25 delivery trucks and calculates the sample mean and sample standard deviation for the cost of operation. A 95% confidence interval for the population mean cost is constructed and found to be $253 to $320. He reasons that this interval contains the mean operating cost for the entire fleet of delivery trucks since the sample mean is contained in this interval.

Required:
a. Do you agree with his reasoning?
b. How would you interpret this confidence interval?
c. Is this an appropriate use of a confidence interval?
d. Concerning your answers to parts a through c, what assumptions did you make (if any)? Does the Central Limit Theorem apply? Why or why not?
e. How does this apply or could apply to your profession?

Answers

Answer:

a) No.

b) Lies between 253 and 320.

c) No.

d) Central limit theorem is not applicable here.

e) The process of estimation can always help us make the right prediction about future sales, demands, costs, profits, and so on based on the past data or any such data available and hence take the correct decision

Step-by-step explanation:

1) No I do not agree, it's just because the mean should be always within the confidence interval. in this case, the above statement doesn't satisfy the condition.  

2)The true mean values at 95% confidence interval lies between 253 and 320.  

3) No, because this is only used for the difference in mean. If there is a difference then the mean is different.  

4) Central limit theorem is not applicable here. Since the sample size is very small, it better to use t distribution rather which indicated normal data.  

in case the sample size is greater than 30, we can then apply the central limit theorem.

5) The process of estimation can always help us make the right prediction about future sales, demands, costs, profits, and so on based on the past data or any such data available and hence take the correct decision.

số 37 không có ước nào là số nguyên tố
nhận xét trên đúng hay sai ?

Answers

Chào Đứa bé!!

Được thôi, nếu vậy câu trả lời cho của bạn câu hỏi là có/thật

I hope it helps u!!

Just comment if u have any questions :-)

Ricky is playing a racing game on PS5.
• He earns 20 points every 1 level he passes.
• He earns 24 points for every 15 laps.
• He drove 45 laps and passed 6 levels.
How many points did Ricky earn?

Answers

Answer:

192 points

Step-by-step explanation:

he earned 120 points for passing 6 levels and getting 20 points per level

he earned 72 points for racing 45 laps and getting 24 points per every 15 laps

5. In ALMN if mL = 48° and mZN = 24", then
O ALMN is an obtuse isosceles triangle
O ALMN is an acute scalene triangle
O ALMN is an obtuse scalene triangle
O ALMN is an acute right triangle
None of the above

Answers

9514 1404 393

Answer:

  (c)  ΔLMN is an obtuse scalene triangle

Step-by-step explanation:

The two given angles are different measures and total less than 90°. That means the third angle will be more than 90°, an obtuse angle. Since all of the angles are different, the triangle is scalene.

  ΔLMN is an obtuse scalene triangle

HELP ME PLSSSS!!!
f(x) = x2 What is g(x)?

A. g(x) = 2x²
B. g(x)=1/2x2
C. g(x)=(1/2x)2
D. g(x)=1/4x2

Answers

Answer:

B. g(x)=1/2x2

Step-by-step explanation:

Function f(x):

[tex]f(x) = x^2[/tex]

At [tex]x = 2, y = f(2) = 2^2 = 4[/tex]

Function g:

Function g is a division of function f by a value. When [tex]x = 2, y = 2[/tex], that is, the value of f is divided by two, and thus, the correct answer is given by option b.

what is the product of -3y²(-4y³+y-9)​

Answers

Answer:

-3y²(-4y³+y-9)​

-9y(-63y-9)

567y-81y

4 8 6 y

please help me and explain

Answers

Answer:

y = 2x + 1

Step-by-step explanation:

the line passes (0, 1) and (2, 5)

the slope = (5-1)/(2-0) = 4/2 = 2

the equation :

y - 1 = 2( x -0)

y-1 = 2x

=> y = 2x + 1

Total employment for sheet metal workers in 2016 is projected to be 201,000. If this is a 6.3% increase from 2006,
approximately what was the tctal employment in 2006?
a. 214,000
b. 187,000
C. 195,000
d. 189,000

Answers

Answer:

D 189,000

Step-by-step explanation:

Other Questions
-3x2 + 5x- 7 = 0 is not in general form.TrueFalse You are baking a cake. The recipe asks for \frac{3}{5} 5 3 cup of butter and you want to make \frac{1}{5} 5 1 of the original recipe. How many cups of butter will you need? describe how lyophobic sols are synthesize by dispersion method 6. Which of these contain muscles that are not under the mind's control? (Select all that apply.) B. The difference between the cash price and the initial deposit in hire purchase is known as ? The potential solutions to the radical equation are a = 4 and a = 1. Which statement is true about these solutions? The solution a = 4 is an extraneous solution. The solution a = 1 is an extraneous solution. Both a = 4 and a = 1 are true solutions. Neither a = 4 nor a = 1 are true solutions. Which position correctly describes the human anatomical position Your school has been performing poorly in public examination for the past three years. Write a letter to your principal suggesting of making the students more interested in academic work. When heat is applied to 80 grams of CaCO3, it yields 39 grams of B. Determine the percentage of the yield. how many solutions does this system have? list 5 uses of computer COUNTY ATTORNEY (with the gallantry of a young politician). And yet, for all their worries, what would we do without the ladies? (The women do not unbend. He goes to the sink, takes a dipperful of water from the pail and, pouring it into a basin, washes his hands. Starts to wipe them on the roller-towel, turns it for a cleaner place.) Dirty towels! (Kicks his foot against the pans under the sink.) Not much of a housekeeper, would you say, ladies? The excerpt contributes most to which feminist theme? Which expression is equivalent to 27 8n? N=0 hurry please my life depends on it work out the size of angle x. For each example presented below, identify the managerial skill being illustrated.a. Charles didn't get much done today because he wasted a lot of time worrying that his coworker was going to gossip about him to their supervisor. b. Although you usually believe that personality tests are not useful in hiring contexts, you feel they are appropriate in this case because the interviewer took the time to explain why the organization uses them and how they will be used, c. You contributed more than of the material for the project for which your team got a performance bonus, but you only received 25% of the bonus amount. A job candidate you are interviewing acts nervous, gives a weak handshake, and fails to talk with confidence. Although the rest of the interview goes well and the candidate seems to have the characteristics you are looking for you give mediocre ratings because of the poor start to the interview.1. First impression bias 2. The halo effect 3. The contrast effect 4. Selective perception Calista shows a pervasive pattern of social deficits and has a great deal of discomfort in close relationships. Her coworkers describe her as odd as she speaks in a peculiar manner, is often suspicious, and is inappropriate in her display of emotion. However, a diagnosis reveals no psychosis. Calista is most likely suffering from ________ disorder. Given the quadriceps function ,y=ax2+bc+c what happens to the graph when "a" is a positive? A TV has a standby current of 0.05 amperes. The voltage going through the TV is 120 volts. Determine the power consumption while the TV is in standby. A conducting sphere of radius R carries an excess positive charge and is very far from any other charges. Draw the graphs that best illustrates the potential (relative to infinity) produced by this sphere as a function of the distance r from the center of the sphere? so L x W=A so I have a problem where one side is 9cm on the other sides there's nothing